Last visit was: 25 Apr 2024, 06:41 It is currently 25 Apr 2024, 06:41

Close
GMAT Club Daily Prep
Thank you for using the timer - this advanced tool can estimate your performance and suggest more practice questions. We have subscribed you to Daily Prep Questions via email.

Customized
for You

we will pick new questions that match your level based on your Timer History

Track
Your Progress

every week, we’ll send you an estimated GMAT score based on your performance

Practice
Pays

we will pick new questions that match your level based on your Timer History
Not interested in getting valuable practice questions and articles delivered to your email? No problem, unsubscribe here.
Close
Request Expert Reply
Confirm Cancel
SORT BY:
Kudos
Tags:
Show Tags
Hide Tags
VP
VP
Joined: 30 Jan 2016
Posts: 1232
Own Kudos [?]: 4560 [14]
Given Kudos: 128
Send PM
Most Helpful Reply
Tutor
Joined: 16 Oct 2010
Posts: 14822
Own Kudos [?]: 64911 [2]
Given Kudos: 426
Location: Pune, India
Send PM
General Discussion
Intern
Intern
Joined: 17 Sep 2018
Posts: 36
Own Kudos [?]: 25 [2]
Given Kudos: 76
Send PM
Manager
Manager
Joined: 14 Oct 2015
Posts: 209
Own Kudos [?]: 345 [0]
Given Kudos: 854
GPA: 3.57
Send PM
Re: A significant amount of the acquisition budget of a typical university [#permalink]
Akela wrote:
A significant amount of the acquisition budget of a typical university library is spent on subscriptions to scholarly journals. Over the last several years, the average subscription rate a library pays for such a journal has increased dramatically, even though the costs of publishing a scholarly journal have remained fairly constant. Obviously, then, in most cases publishing a scholarly journal must be much more profitable now than it was several years ago.

Which one of the following, if true, most seriously weakens the argument?

(A) Many university libraries have begun to charge higher and higher fines for overdue books and periodicals as a way of passing on increased journal subscription costs to library users.
(B) A university library's acquisition budget usually represents only a small fraction of its total operating budget.
(C) Publishing a scholarly journal is an expensive enterprise, and publishers of such journals cannot survive financially if they consistently lose money.
(D) Most subscribers to scholarly journals are individuals, not libraries, and the subscription rates for individuals have generally remained unchanged for the past several years.
(E) The majority of scholarly journals are published no more than four times a year.

Source: LSAT


It should be D

Conclusion: Publishing a scholarly journal must be much more profitable now.

(A) Out of scope/unrelated.
(B) Facts: It is a significant percentage of the acquisition budget, and a small fraction of the operating budget. However we do not know for sure how significant its impact could be for the publishing companies.
(C) This is common wisdom and not specifically targeting the conclusion. Nothing in the argument says publishers of scholarly journal are losing money.
(D) If most of the subscribers are individual, even if rates have gone up for universities, it would not make the journals 'much more profitable' as the conclusion says. This directly goes against the conclusion and thus the correct option.
(E) Not directly related to the conclusion.
Manager
Manager
Joined: 12 Jul 2017
Posts: 199
Own Kudos [?]: 212 [0]
Given Kudos: 442
Location: India
Schools: ISB '21 (A)
GMAT 1: 570 Q43 V26
GMAT 2: 690 Q50 V32
GPA: 3.8
Send PM
Re: A significant amount of the acquisition budget of a typical university [#permalink]
Awesome question!!
Based on weakening an assumption and breaking the "NECESSARY" Condition as per the "must" in the conclusion.
A similar pattern GMAT official question is present and that is:

https://gmatclub.com/forum/music-critic ... 42648.html

Posted from my mobile device
CEO
CEO
Joined: 07 Mar 2019
Posts: 2553
Own Kudos [?]: 1813 [0]
Given Kudos: 763
Location: India
WE:Sales (Energy and Utilities)
Send PM
Re: A significant amount of the acquisition budget of a typical university [#permalink]
A significant amount of the acquisition budget of a typical university library is spent on subscriptions to scholarly journals. Over the last several years, the average subscription rate a library pays for such a journal has increased dramatically, even though the costs of publishing a scholarly journal have remained fairly constant. Obviously, then, in most cases publishing a scholarly journal must be much more profitable now than it was several years ago.

Which one of the following, if true, most seriously weakens the argument?

(A) Many university libraries have begun to charge higher and higher fines for overdue books and periodicals as a way of passing on increased journal subscription costs to library users. - WRONG. Irrelevant.
(B) A university library's acquisition budget usually represents only a small fraction of its total operating budget. - WRONG. Then what!! Does it help us know how profitability at publishing house turn out to be? No.
(C) Publishing a scholarly journal is an expensive enterprise, and publishers of such journals cannot survive financially if they consistently lose money. - WRONG. True in reality but does not help.
(D) Most subscribers to scholarly journals are individuals, not libraries, and the subscription rates for individuals have generally remained unchanged for the past several years. - CORRECT. Yes this looks great. If individuals make a majority then prices have to be kept low which are compensated by university libreries.
(E) The majority of scholarly journals are published no more than four times a year. - WRONG. Could have been a considered had it chosen how the number of times of publishing helps reveal profitability of the publishing house.

Answer D.
GMAT Club Bot
Re: A significant amount of the acquisition budget of a typical university [#permalink]
Moderators:
GMAT Club Verbal Expert
6920 posts
GMAT Club Verbal Expert
238 posts
CR Forum Moderator
832 posts

Powered by phpBB © phpBB Group | Emoji artwork provided by EmojiOne